Post-class edits
This commit is contained in:
parent
e6802de157
commit
09dbcb32a4
@ -6,8 +6,8 @@ We would like to compute the probability of our particle stopping at node $A$. \
|
||||
|
||||
\vspace{2mm}
|
||||
|
||||
In other words, we want a function $P(n): N \to [0, 1]$ that returns the probability that our particle stops at $A$,
|
||||
where $N$ is the set of nodes in $G$.
|
||||
In other words, we want a function $P: \text{Nodes} \to [0, 1]$ that maps each node of the graph
|
||||
to the probability that our particle stops at $A$.
|
||||
|
||||
\begin{center}
|
||||
\begin{tikzpicture}
|
||||
@ -92,11 +92,17 @@ Find $P(x)$ in terms of $P(v_1), P(v_2), ..., P(v_n)$.
|
||||
|
||||
|
||||
\problem{}
|
||||
How can we use \ref{oneunweighted} to find $P(n)$ for any $n$?
|
||||
In general, how do we find $P(n)$ for any node $n$?
|
||||
|
||||
\begin{solution}
|
||||
If we write an equation for each node other than $A$ and $B$, we have a system of $|N| - 2$
|
||||
linear equations in $|N| - 2$ variables.
|
||||
|
||||
\vspace{2mm}
|
||||
|
||||
We still need to show that this system is nonsingular, but
|
||||
that's outside the scope of this handout. This could
|
||||
be offered as a bonus problem.
|
||||
\end{solution}
|
||||
|
||||
\vfill
|
||||
|
@ -1,10 +1,12 @@
|
||||
\section{Circuits}
|
||||
|
||||
An \textit{electrical circuit} is a graph with a few extra properties,
|
||||
called \textit{current}, \textit{voltage}, and \textit{resistance}.
|
||||
called \textit{current}, \textit{voltage}, and \textit{resistance}. \par
|
||||
In the definitions below, let $X$ be the set of nodes in a circuit.
|
||||
|
||||
|
||||
\begin{itemize}[itemsep=3mm]
|
||||
\item \textbf{Voltage} is a function $V(n): N \to \mathbb{R}$ that assigns a number to each node of our graph. \par
|
||||
\item \textbf{Voltage} is a function $V: X \to \mathbb{R}$ that assigns a number to each node of our graph. \par
|
||||
In any circuit, we pick a \say{ground} node, and define the voltage\footnotemark{} there as 0. \par
|
||||
We also select a \say{source} node, and define its voltage as 1. \par
|
||||
|
||||
@ -15,15 +17,18 @@ called \textit{current}, \textit{voltage}, and \textit{resistance}.
|
||||
\footnotetext{
|
||||
In the real world, voltage is always measured \textit{between two points} on a circuit.
|
||||
Voltage is defined as the \textit{difference} in electrical charge between two points.
|
||||
Here, all voltages are measured with respect to our \say{ground} node.
|
||||
Hence, voltage is a function of two nodes.
|
||||
|
||||
This detail isn't directly relevant to the problems in this handout, so you mustn't worry about it today. \par
|
||||
Just remember that the electrical definitions here are a significant oversimplification of reality.
|
||||
\vspace{2mm}
|
||||
|
||||
Note that this is different than current and resistance, which aren't functions
|
||||
of two arbitrary nodes --- rather, they are functions of \textit{edges}
|
||||
(i.e, two adjecent nodes).
|
||||
}
|
||||
|
||||
|
||||
\item \textbf{Current} is a function $I(e^\rightarrow): N \times N \to \mathbb{R}$ that assigns a number to each
|
||||
\textit{oriented edge} $e^\rightarrow$ in our graph. An \say{oriented edge} is just an ordered pair of nodes $(n_1, n_2)$. \par
|
||||
\item \textbf{Current} is a function $I: X^2 \to \mathbb{R}$ that assigns a number to each
|
||||
\textit{oriented edge} in our graph. An \say{oriented edge} is just an ordered pair of nodes $(n_1, n_2)$. \par
|
||||
|
||||
\vspace{1mm}
|
||||
|
||||
@ -31,7 +36,7 @@ called \textit{current}, \textit{voltage}, and \textit{resistance}.
|
||||
Naturally, $I(a, b) = -I(b, a)$.
|
||||
|
||||
|
||||
\item \textbf{Resistance} is a function $R(e): N \times N \to \mathbb{R}^+_0$ that represents a certain edge's
|
||||
\item \textbf{Resistance} is a function $R: X^2 \to \mathbb{R}^+_0$ that represents a certain edge's
|
||||
resistance to the flow of current through it. \par
|
||||
Resistance is a property of each \textit{link} between nodes, so order doesn't matter: $R(a, b) = R(b, a)$.
|
||||
\end{itemize}
|
||||
@ -50,10 +55,14 @@ the case! Any circuit obeys \textit{Ohm's law}, stated below:
|
||||
$$
|
||||
V(a, b) = I(a,b) \times R(a,b)
|
||||
$$
|
||||
\note{
|
||||
$V(a, b)$ is the voltage between nodes $a$ and $b$. If this doesn't make sense, read the footnote below. \\
|
||||
In this handout, it will be convenient to write $V(a, b)$ as $V(a) - V(b)$.
|
||||
}
|
||||
|
||||
This handout uses two notations for voltage: two-variable $V(a, b)$ and one-variable $V(a)$. \par
|
||||
The first represents the voltage between points $a$ and $b$, better reflecting reality (see the footnote below).
|
||||
The second measures the voltage between $a$ and ground, and is more convenient to use in equations.
|
||||
\textbf{Try to use the single-variable notation in your equations.}
|
||||
Convince yourself that $V(a, b) = V(a) - V(b)$.
|
||||
|
||||
\vfill
|
||||
|
||||
|
||||
\definition{Kirchoff's law}
|
||||
@ -64,25 +73,46 @@ Formally, we can state this as follows:
|
||||
|
||||
\vspace{2mm}
|
||||
|
||||
Let $x$ be a node in our circuit and $B_x$ the set of its neighbors. We than have
|
||||
Let $x$ be a node in our circuit and $N_x$ the set of its neighbors. We than have
|
||||
$$
|
||||
\sum_{b \in B_x} I(x, b) = 0
|
||||
\sum_{b \in N_x} I(x, b) = 0
|
||||
$$
|
||||
which must hold at every node \textbf{except the source and ground vertices.} \par
|
||||
\hint{Keep this exception in mind, it is used in a few problems later on.}
|
||||
|
||||
|
||||
\vfill
|
||||
\pagebreak
|
||||
|
||||
|
||||
|
||||
\begin{instructornote}
|
||||
Be aware that some students may not be comfortable with these concepts from physics,
|
||||
nor with the circuit notation on the next page.
|
||||
|
||||
\vspace{2mm}
|
||||
|
||||
It may be a good idea to give the class a quick lecture on this topic,
|
||||
explaining the basics of electonic circuits and circuit diagrams.
|
||||
|
||||
\vspace{2mm}
|
||||
|
||||
Things to cover:
|
||||
\begin{itemize}
|
||||
\item All the definitions on the previous page, in detail.
|
||||
\item What's an Ohm, an Amp, a Volt?
|
||||
\item Measuring voltage. Why is $V(a, b) = V(a) - V(b)$?
|
||||
\item What does the $\Omega$ in the picture below mean?
|
||||
\item Circuit symbols in the diagram below.
|
||||
\end{itemize}
|
||||
|
||||
\vspace{2mm}
|
||||
|
||||
You could also draw connections to the graph flow handout,
|
||||
if the class covered it before.
|
||||
\end{instructornote}
|
||||
|
||||
|
||||
|
||||
|
||||
|
||||
Consider the circuit below. This the graph from \ref{firstgraph}, turned into a circuit by:
|
||||
Consider the circuit below. \textbf{This the graph from \ref{firstgraph}}, turned into a circuit by:
|
||||
\begin{itemize}
|
||||
\item Replacing all edges with $1\Omega$ resistors
|
||||
\item Attaching a 1 volt battery between $A$ and $B$
|
||||
@ -107,7 +137,8 @@ It exists only to create a potential difference between the two nodes.
|
||||
|
||||
\problem{}<onecurrents>
|
||||
From the circuit diagram above, we immediatly know that $V(A) = 1$ and $V(B) = 0$. \par
|
||||
What equations related to the currents out of $x$ and $y$ does Kirchoff's law give us?
|
||||
What equations related to the currents out of $x$ and $y$ does Kirchoff's law give us? \par
|
||||
\hint{Current into $x$ = current out of $x$}
|
||||
|
||||
\vfill
|
||||
|
||||
|
@ -22,12 +22,12 @@ $$
|
||||
|
||||
\problem{}
|
||||
Let $x$ be a node in a graph. \par
|
||||
Let $B_x$ be the set of $x$'s neighbors, $w(x, y)$ the weight of the edge between nodes $x$ and $y$, and $W_x$
|
||||
Let $N_x$ be the set of $x$'s neighbors, $w(x, y)$ the weight of the edge between nodes $x$ and $y$, and $W_x$
|
||||
the sum of the weights of all edges connected to $x$.
|
||||
|
||||
We saw earlier that the probability function $P$ satisfies the following sum:
|
||||
$$
|
||||
P(x) = \sum_{b \in B_x} \biggl( P(b) \times \frac{w(x, b)}{W_x} \biggr)
|
||||
P(x) = \sum_{b \in N_x} \biggl( P(b) \times \frac{w(x, b)}{W_x} \biggr)
|
||||
$$
|
||||
|
||||
\note{This was never explicitly stated, but is noted in \ref{weightedgraph}.}
|
||||
@ -36,7 +36,7 @@ $$
|
||||
|
||||
Use Ohm's and Kirchoff's laws to show that the voltage function $V$ satisfies a similar sum:
|
||||
$$
|
||||
V(x) = \sum_{b \in B_x} \biggl( V(b) \times \frac{C(x, b)}{C_x} \biggr)
|
||||
V(x) = \sum_{b \in N_x} \biggl( V(b) \times \frac{C(x, b)}{C_x} \biggr)
|
||||
$$
|
||||
where $C(x, b)$ is the conductance of edge $(x, b)$ and $C_x$ is the sum of the conductances of all edges connected to $x$.
|
||||
|
||||
@ -44,16 +44,16 @@ where $C(x, b)$ is the conductance of edge $(x, b)$ and $C_x$ is the sum of the
|
||||
\begin{solution}
|
||||
First, we know that
|
||||
$$
|
||||
\sum_{b \in B_x} I(x, b) = 0
|
||||
\sum_{b \in N_x} I(x, b) = 0
|
||||
$$
|
||||
for all nodes $x$. Now, substitute $I(x, b) = \frac{V(x) - V(b)}{R(x, y)}$ and pull out $V(x)$ terms to get
|
||||
$$
|
||||
V(x) \sum_{b \in B_x} \frac{1}{R(x, b)} - \sum_{b \in B_x} \frac{V(b)}{R(x, b)} = 0
|
||||
V(x) \sum_{b \in N_x} \frac{1}{R(x, b)} - \sum_{b \in N_x} \frac{V(b)}{R(x, b)} = 0
|
||||
$$
|
||||
|
||||
Rearranging and replacing $R(x, b)^{-1}$ with $C(x, b)$ and $\sum C(x, b)$ with $C_x$ gives us
|
||||
$$
|
||||
V(x) = \sum_{b \in B_x} V(b) \frac{C(x, b)}{C_x}
|
||||
V(x) = \sum_{b \in N_x} V(b) \frac{C(x, b)}{C_x}
|
||||
$$
|
||||
\end{solution}
|
||||
|
||||
@ -71,7 +71,7 @@ two problems.
|
||||
\problem{}<generaleq>
|
||||
Let $q$ be a solution to the following equations, where $x \neq a, b$.
|
||||
$$
|
||||
q(x) = \sum_{b \in B_x} \biggl( q(b) \times \frac{w(x, b)}{W_x} \biggr)
|
||||
q(x) = \sum_{b \in N_x} \biggl( q(b) \times \frac{w(x, b)}{W_x} \biggr)
|
||||
$$
|
||||
Show that the maximum and minimum of $q$ are $q(a)$ and $q(b)$ (not necessarily in this order).
|
||||
|
||||
@ -80,7 +80,7 @@ Show that the maximum and minimum of $q$ are $q(a)$ and $q(b)$ (not necessarily
|
||||
|
||||
\vspace{2mm}
|
||||
|
||||
Since $q(x)$ is a weighted average of all $q(b), ~b \in B_x$, there exist $y, z \in B_x$ satisfying
|
||||
Since $q(x)$ is a weighted average of all $q(b), ~b \in N_x$, there exist $y, z \in N_x$ satisfying
|
||||
$q(y) \leq q(x) \leq q(z)$. Therefore, none of these can be an extreme point.
|
||||
|
||||
\vspace{2mm}
|
||||
@ -104,7 +104,7 @@ and that $p(x) - q(x) = 0$ for every $x$. \note{Note that $p(x) - q(x) = 0 ~ \fo
|
||||
\begin{solution}
|
||||
The equations in \ref{generaleq} for $p$ and $q$ directly imply that
|
||||
$$
|
||||
[p - q](x) = \sum_{b \in B_x} \biggl( [p - q](b) \times \frac{w(x, b)}{W_x} \biggr)
|
||||
[p - q](x) = \sum_{b \in N_x} \biggl( [p - q](b) \times \frac{w(x, b)}{W_x} \biggr)
|
||||
$$
|
||||
Which are the equations from \ref{generaleq} for $(p - q)$.
|
||||
|
||||
|
@ -38,20 +38,20 @@ out of $A$ is equal to the current flowing into $B$.
|
||||
\problem{}
|
||||
Using Kirchoff's law, show that the following equality holds. \par
|
||||
Remember that we assumed Kirchoff's law holds only at nodes other than $A$ and $B$. \par
|
||||
\note[Note]{As before, $B_x$ is the set of neighbors of $x$. Naturally, $B_B$ is the set of neighbors of $B$.}
|
||||
\note[Note]{As before, $N_x$ is the set of neighbors of $x$.}
|
||||
$$
|
||||
\sum_{b \in B_A} I(S, b) = \sum_{b \in B_B} I(b, B)
|
||||
\sum_{b \in N_A} I(A, b) = \sum_{b \in N_B} I(b, B)
|
||||
$$
|
||||
|
||||
\begin{solution}
|
||||
Add Kirchoff's law for all vertices $x \neq A$ to get
|
||||
$$
|
||||
\sum_{\forall x} \biggl( ~ \sum_{b \in B_x } I(x, b) \biggr) = 0
|
||||
\sum_{\forall x} \biggl( ~ \sum_{b \in N_x } I(x, b) \biggr) = 0
|
||||
$$
|
||||
This sum counts both $I(x, y)$ and $I(x, y)$ for all edges $x, y$, except $I(x, y)$ when $x$ is
|
||||
$A$ or $B$. Since $I(a, b) + I(b, a) = 0$, these cancel out, leaving us with
|
||||
$$
|
||||
\sum_{b \in B_A} I(A, b) + \sum_{b \in B_B} I(B, b) = 0
|
||||
\sum_{b \in N_A} I(A, b) + \sum_{b \in N_B} I(B, b) = 0
|
||||
$$
|
||||
|
||||
\vspace{2mm}
|
||||
@ -61,7 +61,7 @@ $$
|
||||
|
||||
\vfill
|
||||
|
||||
If we call this current $I_A = \sum_{b \in B_A} I(A, b)$, we can pretend that the box contains only one resistor,
|
||||
If we call this current $I_A = \sum_{b \in N_A} I(A, b)$, we can pretend that the box contains only one resistor,
|
||||
carrying $I_A$ units of current. Using this information and Ohm's law, we can calculate the
|
||||
\textit{effective resistance} of the box.
|
||||
|
||||
|
Loading…
x
Reference in New Issue
Block a user